Re: [obm-l] Problema estranho

2017-07-11 Por tôpico Ralph Teixeira
arbitrariamente grandes de 2 Pode isso, Arnaldo? Bom, pode, mas só tem um jeito -- são todos 0. 2017-07-11 18:01 GMT-03:00 Ralph Teixeira : > Bom, eu sei resolver se todos os números forem racionais. Deve ter um > jeito de usar isso para o caso geral... > > A propriedade desse con

Re: [obm-l] Problema estranho

2017-07-11 Por tôpico Ralph Teixeira
Bom, eu sei resolver se todos os números forem racionais. Deve ter um jeito de usar isso para o caso geral... A propriedade desse conjunto não se altera se todos os elementos do conjunto forem multiplicados por um mesmo número, nem se a gente somar uma certa constante a todos eles. Assim, *SE* el

Re: [obm-l] Contra-positiva

2017-06-21 Por tôpico Ralph Teixeira
Sim! Logicamente, P->Q eh exatamente a mesma coisa que ~Q->~P, pode trocar uma pela outra sem pestanejar. Como voce estah trocando algo por outra coisa EQUIVALENTE, nao precisa fazer nenhum outro ajuste. Entao, se a frase eh, "Para todo x real, existe um y inteiro tal que para todo z par existe u

[obm-l] Re: [obm-l] Indução forte vs fraca

2017-06-17 Por tôpico Ralph Teixeira
Sim, isso eh uma demonstracao valida por inducao finita! Na minha opiniao, nem precisa formalizar muito mais -- a IDEIA da inducao como voce propos eh bastante comum, entao dah para escrever direto algo como voce disse, assim: a) Provo P(1) e P(2); b) Provo que P(k-1) e P(k) implicam P(k+1) para k

Re: [obm-l] Convexidade

2017-05-23 Por tôpico Ralph Teixeira
Oi, Israel. No livro "Analise Real, Volume 1" do nosso saudoso Elon ( :( ), tem na Seção 9.2 (Funções Convexas e Concavas) uma discussão bem cuidadosa sobre isso. Depois de definir "convexidade" via aquela desigualdade, ele prova 3 teoremas; o primeiro não supõe nada adicional sobre f, o segund

Re: [obm-l] Re: Menor caminho

2017-03-04 Por tôpico Ralph Teixeira
Minha solucao favorita eh bem geometrica. Vamos procurar um caminho AYXB onde Y estah no eixo Oy e X estah no eixo Ox. Considere C(-3,13), obtido refletindo A em torno do eixo Oy; e D(9,-3), obtido rebatendo B em torno de Ox. A chave eh a seguinte: qualquer que seja o caminho AYXB que voce tomar,

[obm-l] Re: [obm-l] Re: [obm-l] Re: [obm-l] Intervalos de crescimento de uma função.

2017-02-22 Por tôpico Ralph Teixeira
de fev de 2017 7:51 PM, "Ralph Teixeira" escreveu: >> >> Bom, nao tem dados na sua pergunta, mas concordo que muita gente >> confunde as frases: >> >> 1. "A funcao eh crescente em [-4,-3] e em [2,3]." >> e >> 2. "A funcao eh cresce

[obm-l] Re: [obm-l] Intervalos de crescimento de uma função.

2017-02-22 Por tôpico Ralph Teixeira
Bom, nao tem dados na sua pergunta, mas concordo que muita gente confunde as frases: 1. "A funcao eh crescente em [-4,-3] e em [2,3]." e 2. "A funcao eh crescente em [-4,-3]U[2,3]." que, na minha modesta opiniao, NAO significam a mesma coisa... Pode ser que a primeira seja verdadeira mas a segund

[obm-l] Re: [obm-l] Função f(n) = (1 + 1/n)^n é crescente?

2016-12-25 Por tôpico Ralph Teixeira
Vou indicar esqueleto de argumentos: Opcao 1: Use M.A.>=M.G. com os numeros 1, 1+1/n, 1+1/n,... 1+1/n (com n copias desse ultimo). Opcao 2: Fazendo contas, vem f(n+1)/f(n) = (1+1/n).(1-1/(n+1)^2)^(n+1). Agora, Bernoulli diz que (1+x)^n > 1+nx quando x>-1 (x<>0) e n>=2 (mostre isso usando inducao

Re: [obm-l] Combinatoria

2016-12-23 Por tôpico Ralph Teixeira
, 1) > (4, 4, 2) > (4, 2, 4) > São 8 combinações ao todo. Como um cubo 8x8x8 pode ser montado com 4 cubos > 4x4x4, poderíamos certamente cobrir todas as possibilidades com 4*8 = 32 > tentativas. Portanto, a resposta é menor ou igual a 32. > > > Em 23 de dezembro de 2016 16:22, Ralph

Re: [obm-l] Combinatoria

2016-12-23 Por tôpico Ralph Teixeira
Hm, acho que dah para fazer com menos tentativas. Sejam a, b e c as combinacoes corretas de cada cadeado, onde a,b,c estao em {0,1,2,3,4,5,6,7}. Tentanto, por exemplo, todas as combinacoes possiveis para a e b (mantenha c=0), fazemos 64 tentativas, e com certeza vamos acabar acertando a combinaca

Re: [obm-l] Irracionalidade

2016-11-24 Por tôpico Ralph Teixeira
Nao, porque a soma deles eh constante e igual a pi/2. 2016-11-23 21:29 GMT-02:00 Israel Meireles Chrisostomo < israelmchrisost...@gmail.com>: > É possível encontrar x tal que arccot(x) seja racional e arccot(1/x) seja > racional? > > -- > Esta mensagem foi verificada pelo sistema de antivírus e >

[obm-l] Re: [obm-l] Re: [obm-l] Re: [obm-l] Sequências

2016-11-16 Por tôpico Ralph Teixeira
nda parte do > problema Ralph? > > Em 16 de novembro de 2016 22:09, Ralph Teixeira > escreveu: > >> Dica para comecar: se A_k={a,b,c,x} onde x eh a media de a,b e c, o que >> voce pode dizer sobre a soma dos elementos de A_k? >> >> Abraco, Ralph. >> >

[obm-l] Re: [obm-l] Sequências

2016-11-16 Por tôpico Ralph Teixeira
Dica para comecar: se A_k={a,b,c,x} onde x eh a media de a,b e c, o que voce pode dizer sobre a soma dos elementos de A_k? Abraco, Ralph. 2016-11-16 21:58 GMT-02:00 Pedro Júnior : > Ainda não consegui esse problema. Ele foi do livro do Caminha. > Ache todos os valores de $n$ para os quais possam

[obm-l] Re: [obm-l] Re: [obm-l] Re: [obm-l] aritmética

2016-10-13 Por tôpico Ralph Teixeira
Hm, devagar -- por exemplo (4,2)=6 nao eh multiplo de 4. Abraco, Ralph. 2016-10-13 17:25 GMT-03:00 Pedro José : > Boa tarde! > > Basta que p seja diferente de 0 ou n, para n<>0. > > (n,p) = n! / (p!. (n-p)!), > > Portanto, só há como tirar o fator n do n! se p! = n! ou (n-p)! = n! ==> p > = 0 ou

Re: [obm-l] Problema

2016-10-08 Por tôpico Ralph Teixeira
Oi, Regis. Eu acho (acho!) que nao dah para resolver esse sistema no braco com as funcoes elementares usuais. Eliminando uma das variaveis, recai em algo do tipo: ln(lny)+(ln2)/y=ln(ln3) ou ln(lnx)+(ln3)/x=ln(ln2) E, ateh onde eu consigo pensar, equacoes desse tipo nao se resolvem no braco. Ago

Re: [obm-l] Re: Quadrado perfeito?

2016-08-24 Por tôpico Ralph Teixeira
Ah, nao li, mas tem isso: http://djm.cc/dmoews/und.pdf 2016-08-24 9:23 GMT-03:00 Ralph Teixeira : > Acho que eh um problema dificil: > http://mathworld.wolfram.com/UndulatingNumber.html > > 2016-08-24 9:12 GMT-03:00 marcone augusto araújo borges < > marconeborge...@hotmail.co

Re: [obm-l] Re: Quadrado perfeito?

2016-08-24 Por tôpico Ralph Teixeira
Acho que eh um problema dificil: http://mathworld.wolfram.com/UndulatingNumber.html 2016-08-24 9:12 GMT-03:00 marcone augusto araújo borges < marconeborge...@hotmail.com>: > 3535...35 não foi um bom exemplo porque já haviam sido mencionado os > > os ab é da forma 4k+3. > > > -

[obm-l] Re: [obm-l] Re: [obm-l] Re: [obm-l] Re: Números complexos

2016-08-23 Por tôpico Ralph Teixeira
Ah, troquei i por -i em algumas linhas, o que por sorte nao altera a resposta... Mas corrijo abaixo: 2016-08-23 9:39 GMT-03:00 Ralph Teixeira : > Na minha opiniao, a principal "ambiguidade" da sua pergunta seria: qual > das duas voce quer? > > 1) Encontre todos os valores

[obm-l] Re: [obm-l] Re: [obm-l] Re: [obm-l] Re: Números complexos

2016-08-23 Por tôpico Ralph Teixeira
Na minha opiniao, a principal "ambiguidade" da sua pergunta seria: qual das duas voce quer? 1) Encontre todos os valores reais de x tais que (x+i)^(4n) eh real PARA TODO n NATURAL; 2) Encontre todos os valores reais de x tais que (x+i)^(4n) eh real PARA ALGUM n NATURAL; Mas vamos lah: ---///---

[obm-l] Re: [obm-l] [obm-l] Simulado ITA (Equação Modular)

2016-08-22 Por tôpico Ralph Teixeira
Confira as suas contas -- cada uma daquelas 4 equacoes tem uma raiz real valida. Abraco, Ralph. 2016-08-22 16:38 GMT-03:00 Ricardo Leão : > Olá amigos, > > Eu gostaria que algum amigo corrigisse a solução que eu desenvolvi para o > seguinte problema envolvendo módulo: > > (Enunciado) O numero d

[obm-l] Re: [obm-l] Re: [obm-l] Re: [obm-l] Outra dúvida

2016-06-09 Por tôpico Ralph Teixeira
Acho que eles querem que voce pense assim: quanto mais "aberto" eh o cone, maior eh a area lateral. Entao a maior area lateral possivel seria o caso degenerado onde o cone estah tao aberto que eh, de fato, um disco, que teria area pi. Mas eles querem o maior INTEIRO possivel para a area, que seria

Re: [obm-l] Enunciado

2016-04-25 Por tôpico Ralph Teixeira
1) Na equacao original, multiplique tudo pelos conjugados: 1=2(y-sqrt(1+y^2))(x-sqrt(1+x^2)) (y-sqrt(1+y^2))(x-sqrt(1+x^2))=1/2 Agora some isso com a equacao original. Vem xy+sqrt(1+x^2).sqrt(1+y^2)=5/4 sqrt(1+x^2).sqrt(1+y^2)=5/4 - xy Eleve ao quadrado: (1+x^2)(1+y^2)=25/16 -5/2 xy +x^2.y^2 x

[obm-l] Re: [obm-l] Fatorial e números primos

2016-04-01 Por tôpico Ralph Teixeira
Falso. Tome n=3^5 como contra-exemplo. 2016-04-01 18:17 GMT-03:00 Pedro Chaves : > Caros Colegas, > > Proponho o teorema abaixo. > > Teorema: > > --- Na decomposição em fatores primos positivos do inteiro n >3, o fator > 2 aparece mais vezes do que qualquer outro fator. --- > > Agradeço-lhes a

Re: [obm-l] Desigualdade, limitante inferior

2016-03-14 Por tôpico Ralph Teixeira
O mesmo epsilon para todas as escolhas positivas das variaveis? Acho que nao. Seja F(x,y,z,a,b,c)=ax+by+cz restrita ao dominio definido por {xy+xz+yz=1; ab+ac+bc=1}. Note que F eh continua. Agora, dado eps>0, tome k>0 tal que F(k,1/k,0, k, 0, 1/k) = k^2: > Olá pessoal, > Seja xy+xz+yz=1 e x'y'+x

[obm-l] Re: [obm-l] Re: Indução dúvida

2016-01-19 Por tôpico Ralph Teixeira
Sim, a estrutura me parece correta. 2016-01-18 15:47 GMT-02:00 Israel Meireles Chrisostomo < israelmchrisost...@gmail.com>: > Por exemplo, eu quero provar que f(n)>c para todo n inteiro.Então, eu > provei o caso base,e considerei a hipótese de indução, suponha que é válido > para um k que f(k)>c

[obm-l] Re: [obm-l] Indução dúvida

2016-01-18 Por tôpico Ralph Teixeira
Oi, Israel. Realmente muita gente faz essa confusao. Voce quer provar que "Para todo n natural, P(n) eh VERDADEIRA." O metodo de inducao, em sua versao mais simples, diz que basta mostrar duas coisas: i) P(1) eh VERDADEIRA ii) Para todo k natural, (P(k)->P(k+1)). Note com cuidado onde estao o

[obm-l] Re: [obm-l] RE: [obm-l] Re: [obm-l] duas séries e um resultado

2016-01-11 Por tôpico Ralph Teixeira
:00 Ralph Teixeira : > Sim, essa eh uma otima maneira de "consertar" o enunciado... Eu acho que > prefiro mudar um pouquinho a definicao de a_n para ter o mesmo numero de > termos do b_n -- fica mais bonitinho, e nao deve fazer diferenca, pois > minha definicao e a sua vao dif

[obm-l] Re: [obm-l] RE: [obm-l] Re: [obm-l] duas séries e um resultado

2016-01-11 Por tôpico Ralph Teixeira
Sim, essa eh uma otima maneira de "consertar" o enunciado... Eu acho que prefiro mudar um pouquinho a definicao de a_n para ter o mesmo numero de termos do b_n -- fica mais bonitinho, e nao deve fazer diferenca, pois minha definicao e a sua vao diferir de 1/sqrt(2n+1), que tende a 0. Ou seja, quero

[obm-l] Re: [obm-l] duas séries e um resultado

2016-01-11 Por tôpico Ralph Teixeira
raizes dos pares, ficando a soma dos inversos das raizes dos impares, que seria o a. Mas, como eu disse, estah errado -- pelo menos no universo dos reais, b nao existe, nem sqrt(2)b, nem a. Abraco, Ralph. 2016-01-11 17:18 GMT-02:00 Ralph Teixeira : > Bom, se eu entendi, do jeito que estah eh

[obm-l] Re: [obm-l] duas séries e um resultado

2016-01-11 Por tôpico Ralph Teixeira
Bom, se eu entendi, do jeito que estah eh falso, porque nenhuma destas series converge! (Bom, pelo menos nos reais... A menos que eles estejam em algum outro sistema...) Abraco, Ralph. 2016-01-11 12:31 GMT-02:00 Luís : > Sauda,c~oes, > > Um bom 2016 para todos. > > Recebi o seguinte problema. >

[obm-l] Re: [obm-l] Máximo absoluto de f(x) =( 5x -1) / (x^2 + 1)

2015-12-13 Por tôpico Ralph Teixeira
Vamos encontrar a imagem de f(x). Para tanto, escreva f(x)=k, isto eh: 5x-1=k(x^2+1) k.x^2-5x+(k+1)=0 Esta equacao tem raiz real em x se, e somente se, 25-4.k.(k+1)>=0, isto eh, k^2-k-25/4<=0. Ou seja, se [-1-raiz(26)]/2<=k<=[-1+raiz(26)]/2. Entao o maximo da funcao eh [raiz(26)-1]/2 (e o minimo

Re: [obm-l] Geometria Plana

2015-11-26 Por tôpico Ralph Teixeira
Sim, ha um argumento simples e convincente para a existencia de tal triangulo. Comece pela circunferencia de raio R. Marque nela ARCOS consecutivos de comprimento angular 2a, 2b e 2c (como 2a+2b+2c=2pi, o terceiro arco termina onde o primeiro comeca). Use as pontas destes arcos para serem os verti

Re: [obm-l] Primo?

2015-11-24 Por tôpico Ralph Teixeira
Deixa eu ver 2^83-1 2^83-1... Ah, é, 2^83-1... Se eu me lembro bem, vale: 2^83-1 = 167×57912614113275649087721 Confere aí se eu errei algum dígito. ;) ;) ;) ;) 2015-11-24 7:32 GMT-02:00 marcone augusto araújo borges < marconeborge...@hotmail.com>: > Mostre que 2^83 - 1 não é primo > >

Re: [obm-l] Sistema simples

2015-10-23 Por tôpico Ralph Teixeira
Nao. Note que x/(x+1)=u/(u+v+w), y/(y+1)=v/(u+v+w) e z/(z+1)=w/(u+v+w). Entao ha uma restricao: x/(x+1)+y/(y+1)+z/(z+1)=1. Por outro lado, se isso valer, entao sim -- basta tomar u=kx/(x+1), v=ky/(y+1), w=kz/(z+1), onde k eh um real positivo qualquer. Abraco, Ralph. 2015-10-23 21:22 GMT-02:00 I

Re: [obm-l] Sistema

2015-10-23 Por tôpico Ralph Teixeira
Bom, nao funciona -- se x/(y+z) for negativo, voce nao vai achar u, v e w nunca... :( 2015-10-23 16:25 GMT-02:00 Israel Meireles Chrisostomo < israelmchrisost...@gmail.com>: > Olá pessoal estive resolvendo uma desigualdade, e consegui achar uma > segunda solução para essa desigualdade, para prova

[obm-l] Re: [obm-l] relação trigonométrica

2015-10-22 Por tôpico Ralph Teixeira
Se voce partir de (b-c)/(b+c) e usar a lei dos senos (b=2RsinB e c=2RsinC), depois transformar somas em produtos, acho que sai rapido, sem grandes magicas... Nao? Abraco, Ralph. 2015-10-22 9:21 GMT-02:00 Luís : > Sauda,c~oes, > > Considere o triângulo ABC com b>c e o ângulo > D = (B-C)/2. > > Co

[obm-l] Re: [obm-l] Cálculo limite

2015-09-25 Por tôpico Ralph Teixeira
Definicao de derivada? Hm, derivada de que funcao em que ponto? De qualquer forma, aposto que, por algum motivo, estah faltando um "-1" no numerador. Aposto que voce trocou algum f(0) por 0 em algum canto, e que devia ser ao inves: lim (h->0) {[1+(h/x)]^n-1}/(h/x) Serah? Abraco, Ralph. 2015-0

Re: [obm-l] Pergunta que gera debates

2015-09-24 Por tôpico Ralph Teixeira
(Suponho que estamos todos usando "sqrt" no lugar do símbolo usual de raiz quadrada) Pois é... Um dois problemas é que sqrt não é muito bem definida nos complexos... Por exemplo, eu não sei se sqrt(5-12i) dá 3-2i ou -3+2i. Mas você pode adotar uma convenção apropriada. Por exemplo, todo mundo usa

Re: [obm-l] Funcao Injetora

2015-09-03 Por tôpico Ralph Teixeira
Para provar que f eh injetora, basta mostrar que f(x)=f(y) implica x=y. Neste caso, se voce tivesse f(x)=f(y), teria f(f(x))=f(f(y)) (eu posso aplicar f de novo pois o contradominio estah contido no dominio). Mas entao 2x=f(x)+f(f(x))=f(y)+f(f(y))=2y, isto eh, x=y. Abraco, Ralph. 2015-09-03 20:5

[obm-l] Re: [obm-l] Função

2015-08-13 Por tôpico Ralph Teixeira
Tecnicamente, eu diria que f(x)=0 faz o que voce pediu. Mas acho que voce quer algo como f(x)=2x/(1+x^2). Eh facil ver que -1<=f(x)<=1 para todo x real, e os pontos criticos sao atingidos em x=+-1. 2015-08-13 19:10 GMT-03:00 Israel Meireles Chrisostomo < israelmchrisost...@gmail.com>: > É possív

[obm-l] Re: [obm-l] Dúvida "aparentemente" simples

2015-08-11 Por tôpico Ralph Teixeira
Acho que a convencao de quase todos eh que -0=0. Nao vejo problema de que o oposto de algo seja ele mesmo. 2015-08-11 11:27 GMT-03:00 Vanderlei Nemitz : > Trabalho com edição de material didático e outro dia, em conversa com um > autor de material do ensino médio, surgiu uma questão sobre a qual

[obm-l] Re: [obm-l] Sobre mdc de polinômios

2015-08-05 Por tôpico Ralph Teixeira
Oi, Listeiro. A chave eh notar que divisibilidade de polinomios nao eh a mesma coisa que divisibilidade dos numeros que eles representam (quando voce substitui x). Digamos, exagerando, que nao tem nada a ver um com o outro. Um polinomio p(x) eh divisivel por um polinomio D(x) quando pode se escre

Re: [obm-l] Probleminha

2015-07-31 Por tôpico Ralph Teixeira
*aquele primeiro n era S. :) 2015-07-31 16:39 GMT-03:00 Ralph Teixeira : > Note que x e y=1/x sao as raizes da quadratica t^2-nt+1=0, onde S=x+y eh > inteiro. > > Agora escreva S_k=x^k+y^k. Note que: > S_(k+1)=x^2.x^(k-1)+y^2.y^(k-1) = (Sx-1).x^(k-1)+(Sy-1).y^(k-1) = S.S_k - >

Re: [obm-l] Probleminha

2015-07-31 Por tôpico Ralph Teixeira
Note que x e y=1/x sao as raizes da quadratica t^2-nt+1=0, onde S=x+y eh inteiro. Agora escreva S_k=x^k+y^k. Note que: S_(k+1)=x^2.x^(k-1)+y^2.y^(k-1) = (Sx-1).x^(k-1)+(Sy-1).y^(k-1) = S.S_k - S_(k-1) Entao a sequencia {S0, S1, ...} satisfaz esta recorrencia de coeficientes inteiros! Como S_0=2 e

Re: [obm-l] probleminha

2015-07-31 Por tôpico Ralph Teixeira
Note: S(2n) eh divisivel por 9, entao 2n eh divisivel por 9, entao n eh divisivel por 9, entao S(n) eh divisivel por 9, entao S(2n) eh divisivel por 81, entao S(n) eh divisivel por 144. Agora eu vou tentar arrumar algum n que satisfaz esta condicao S(n)=144 e S(2n)=81, para pelo menos ter uma idei

[obm-l] Re: [obm-l] Re: [obm-l] Questão Colégio Militar - anulada

2015-07-26 Por tôpico Ralph Teixeira
P.S.: Oops, erro tipografico: troque aquele "10 minutos" do finalzinho por 15. O resto estah ok: sao, de fato, 11h50m. 2015-07-26 14:18 GMT-03:00 Ralph Teixeira : > Para mim, letra (a). > > O problema eh esquisito porque a lingua portuguesa parece esquisita > Afinal: &g

[obm-l] Re: [obm-l] Re: [obm-l] Questão Colégio Militar - anulada

2015-07-26 Por tôpico Ralph Teixeira
Para mim, letra (a). O problema eh esquisito porque a lingua portuguesa parece esquisita Afinal: Suponha que voce tem um encontro na hora x. Se voce estah ATRASADO y minutos, voce chegou na hora x+y. Se voce estah ADIANTADO y minutos, voce chegou na hora x-y. A principio, a linguagem PARECE

[obm-l] Re: [obm-l] Geometria analítica em Três dimensões

2015-07-23 Por tôpico Ralph Teixeira
Hm, pera, tem 4 variaveis ai. A letra t representa um numero fixo, e as variaveis sao x, y e z? Vou supor que sim, senao eh uma "superficie" em 4 dimensoes. Bom, entao a resposta eh sim, representa. Se esta figura tem nome proprio, bom, ok, nao sei. :) Mas notei que se voce botar x=t.sina, y=t.si

[obm-l] Re: [obm-l] Quantas funções?

2015-07-21 Por tôpico Ralph Teixeira
De uma olhada aqui: https://oeis.org/A019538 Abraco, Ralph. 2015-07-21 20:00 GMT-03:00 marcone augusto araújo borges < marconeborge...@hotmail.com>: > Dados os conjuntos A com n elementos e b com m elementos, quantas > são as funções sobrejetivas de A em B? > > -- > Esta mensagem foi verificada

[obm-l] Re: [obm-l] Re: [obm-l] Re: [obm-l] Re: [obm-l] Princípio das gavetas

2015-07-17 Por tôpico Ralph Teixeira
Considere a1, a1+a2, a1+a2+a3,...,a1+a2+...+a(n-1) e também a2. Se a1<>a2 mod n, e nenhum desses for divisível por n, então dois desses têm o mesmo resto na divisão por n (e não são a1 e a2). Subtraia os dois, acabou. Ou seja, o único contra-exemplo é mesmo a1=a2=...=a(n-1) mod n com (a1,n)=1. A

[obm-l] Re: [obm-l] Re: [obm-l] dúvida

2015-07-09 Por tôpico Ralph Teixeira
Vamos generalizar para R^n: com a noção usual (Euclideana) de comprimento, o comprimento do segmento que liga (x1,x2,...,xn) a (y1,y2,...,yn) é: d=raiz((y1-x1)^2+(y2-x2)^2+...+(yn-xn)^2) Esta é a noção usual de distância entre dois pontos -- confira que é o que você conhece na reta (n=1) e no pla

[obm-l] Re: [obm-l] Re: [obm-l] transcedência

2015-07-08 Por tôpico Ralph Teixeira
A ferramenta que eu conheco para provar isso eh a Teoria de Galois. Eu soh vi isso no curso de Algebra do Mestrado lah no IMPA (nao vi no curso de Algebra da graduacao). Aqui: https://en.wikipedia.org/wiki/Galois_theory Menos completo, em portugues: https://pt.wikipedia.org/wiki/Teoria_de_Galois A

[obm-l] Re: [obm-l] Re: [obm-l] Sistema de equações

2015-07-03 Por tôpico Ralph Teixeira
Bom, podemos mostrar que sen²x+sen²y+sen²z=1; x+y+z=pi/2 implicam que algum dos ângulos x, y, z é múltiplo de pi/2 (em particular, não serão todos positivos). Serve para o que você quer? Em primeiro lugar, tome A=2x, B=2y e C=2z. Traduzimos tudo então para: (1-cosA)/2+(1-cosB)/2+(1-cosC)/2=1, isto

[obm-l] Re: [obm-l] RE: [obm-l] Polinômio

2015-06-29 Por tôpico Ralph Teixeira
Bom, vou dizer como cheguei nisso, mas aposto que minha maneira convoluida de pensar nao eh a melhor -- estah cheia de truques, enquanto dah para fazer tudo isso usando divisibilidade e restos da mesma maneira que usa-se o Teorema Chines dos Restos Mas vamos lah, pensei assim: Como p(x)+1 tem

[obm-l] Re: [obm-l] Polinômio

2015-06-29 Por tôpico Ralph Teixeira
Não falta nada no enunciado? Sem saber algo sobre o grau, p(x) não fica determinado. Basicamente, qualquer coisa do tipo: p(x)=(x^2+1)[(x^3+x^2+1)q(x)+x^2+x-1] serve. Abraço, Ralph. 2015-06-29 10:21 GMT-03:00 marcone augusto araújo borges < marconeborge...@hotmail.com>: > Determine p(x), divis

[obm-l] Re: [obm-l] Funçao periódica

2015-06-12 Por tôpico Ralph Teixeira
Oi, Marcone. Acho que está faltando uma definição precisa de função periódica para você poder trabalhar. Definição: Uma função real f(x) é *periódica* se existe T>0 tal que f(x+T)=f(x) para todo x real. Neste caso, o número T é dito um *período* da função. Se existir um menor número positivo T sa

Re: [obm-l] {Filename?} Problema Interessante de Geometria

2015-06-09 Por tôpico Ralph Teixeira
Oi, Paulo. Mas aqui que estah o problema -- nao eh dado que PiPi+1 e igual a QiQi+1, soh que sao paralelos... :) Abraco, Ralph. 2015-06-09 16:31 GMT-03:00 Paulo Santa Rita : > Ola Fabiola, Prof da Fabiola e carissimo Ralph, > > Vou fazer um esboço de prova aqui. Considere os triângulos OPiPi+1

[obm-l] {Filename?} Problema Interessante de Geometria

2015-06-08 Por tôpico Ralph Teixeira
Warning: Esta mensagem continha anexos que foram removidos Warning: (geogebra_javascript.js, Hexagons.ggb). Warning: Leia o anexo "DMAT-PUCRJ-Attachment-Warning.txt" para maiores informa��es. Ola a todos. Eu e minha aluna de Mestrado Fabiola encontramos um problema bem facil de enunciar que escl

Re: [obm-l] exatamente

2015-06-04 Por tôpico Ralph Teixeira
Para mim, voce acertou, e essa resposta estah errada. Voce escolheu fazer a contagem como se a ordem importasse, o que eh perfeitamente valido. Estamos supondo que o sorteio eh justo, no sentido de que todas as PESSOAS tem a mesma chance de serem sorteadas (e nao todas as PATENTES, Entao, sim, ha

Re: [obm-l] RES: soma finita??? corrigindo

2015-06-02 Por tôpico Ralph Teixeira
Suponho que seja 2^(n-1)*n? Seja 1S = 1.1+2.2+4.3+8.4+...+2^(n-1).n Entao, botando um 0 na frente para alinhar do jeito que eu quero: 2S = 0.0+2.1+4.2+8.3+...+2^(n-1).(n-1)+2^n.n Subtraindo e vendo a PG negativa: S = -1 -2 -4 -8... -2^(n-1) + 2^n.n = 2^n.n - 2^n + 1= 2^n.(n-1) + 1 Divida por n, e

Re: [obm-l] Determinante de uma matriz de coeficientes inteiros

2015-06-02 Por tôpico Ralph Teixeira
A=[10, 7, 2; 8, 10, 8; 2, 8, 10] :) :) :) 2015-06-02 4:36 GMT-03:00 Lucas Colucci : > Bom dia! > > Seja A uma matriz nxn de entradas inteiras positivas e tal que, para todos > índices i e j distintos, a_{i, i}>a_{i , j}, a_{j, i}. Isso implica que det > A é diferente de zero? > > Lucas Colucci >

[obm-l] Re: [obm-l] Dúvida

2015-06-01 Por tôpico Ralph Teixeira
Bom, depende muito do que chamamos de "formas"... Vou supor que as posicoes sao todas importantes e rotuladas. Digo, vou contar como *diferentes* preenchimentos que difiram por rotacao, reflexao ou permutacao dos numeros. Eu comecaria notando que se alguma posicao preenche o quadrado, entao qualqu

[obm-l] Re: [obm-l] Implicação lógica

2015-05-23 Por tôpico Ralph Teixeira
Oi, Pedro. Do jeito que eu estou acostumado a escrever, nao ha diferenca entre --> e =>. Para mim, ambas sao a mesma coisa, ambas significam que "p implica q", e ambas estao corretas. Sim, p:"2 eh impar" implica logicamente que q:"2+2 eh impar" (e implica tambem que "eu sou o Papa", como voce mesm

Re: [obm-l] Contagem de funcoes

2015-05-16 Por tôpico Ralph Teixeira
Seja S={y | f(y)=y}. Entao a condicao eh equivalente a dizer que f(x) \in S para todo x. Em suma, para escolher a funcao f, vou escolher o conjunto S (onde a funcao tem que ser a identidade), e depois escolho os valores de f(x) \in S para os x FORA de S. Dividindo em casos: i) #S=1. Ha 5 escolha

Re: [obm-l] Gavetas

2015-05-10 Por tôpico Ralph Teixeira
Vamos repartir A em 9 conjuntos: A1={1,10,19,28,...,91,100} A2={2,11,20,29,...,92} A3={3,12,21,30,...,93} ... A9=(9,18,27,36,...,99} Como sao 55 numeros escolhidos e 9 conjuntos, pelo menos um conjunto tem pelo menos [55/9]+1=7 numeros escolhidos. (Se cada um tivesse 6 ou menos, teriamos um tota

[obm-l] Re: [obm-l] Pontos numa Circunferência

2015-05-06 Por tôpico Ralph Teixeira
1a) Vou indicar os passos escrevendo os numeros em fila (lembre que o ultimo eh adjacente ao primeiro). Na posicao inicial, temos: 0100... Agora escolha trocar os 1s da esquerda para a direita, assim: 1010... 1101... 11101000... ... ...1010 ...1101 01

Re: [obm-l] Contagem, preciso de uma ajuda!!

2015-05-06 Por tôpico Ralph Teixeira
Vou supor que cumes nao existem nos pontos (1,a) e (10,b). Se cumes nas "pontas" forem validos, o raciocinio tem que mudar um pouquinho. Entao a pergunta eh: quantos caminhos tem um cume no ponto (x,y)? Por exemplo, quantos caminhos tem cume em (2,10)? Ora, para um caminho ter cume em (2,10), bas

[obm-l] Re: [obm-l] Dúvida Análise(tannery's theorem)

2015-05-05 Por tôpico Ralph Teixeira
Não sei se entendi a pergunta também... Mas *talvez* esse seja um exemplo bom... Considere a sequencia dupla a(k,n) (onde k,n=1,2,3,...) dada por: a(k,n) = 1/k se n<=k a(k,n) = 0 se n>k Ou seja, mais explicitamente, colocando k fixo e variando n em cada linha: a(1,n): 1,0,0,0,0,0,0,.. a(2,n): 1

Re: [obm-l] Irracionalidade de pi

2015-05-03 Por tôpico Ralph Teixeira
zeta é igual a pi^2k multiplicado por um > racional, o que só pode ser provado por meio de limites, e então, o que vc > me diz? Logo, acredito que esse seu argumento perde o efeito. > > Em 2 de maio de 2015 17:54, Ralph Teixeira escreveu: > >> Nao funciona... Voce pode ter

Re: [obm-l] Irracionalidade de pi

2015-05-02 Por tôpico Ralph Teixeira
Nao funciona... Voce pode ter uma infinidade de numeros irracionais, cujo limite eh RACIONAL. Pense por exemplo na sequencia a_n = Raiz(2)/n Todos esses a_n sao irracionais, mas o limite da sequencia eh 0, um racional. Ou seja, como voce suspetaiva, soh porque alguma propriedade vale para todo n

Re: [obm-l] irracionalidade

2015-04-29 Por tôpico Ralph Teixeira
Mas eh falso. Tome r=raiz(2)-1 e k=2. 2015-04-29 13:50 GMT-03:00 Israel Meireles Chrisostomo < israelmchrisost...@gmail.com>: > Alguém sabe se é possível provar que:seja k um natural,então se r^k é > irracional então (r+1)^k também é irracional? > > -- > Esta mensagem foi verificada pelo sistema

[obm-l] Re: [obm-l] Re: [obm-l] FW: pentágono cíclico

2015-04-24 Por tôpico Ralph Teixeira
eu método funcionaria > a princípio para qualquer quadrilátero, o que não faz sentido. > > De todo modo, vindo do Luís, acredito que seja um problema > de contrução com régua e compasso. > > Abraço, > Sergio > > On Friday, April 24, 2015, Ralph Teixeira wrote: > >> C

[obm-l] Re: [obm-l] FW: pentágono cíclico

2015-04-24 Por tôpico Ralph Teixeira
Construir, tipo, com regua e compasso? Ou, num sentido mais teorico e geral? Pegue um crculo com raio 300, marque pontos ABCDEF tal que as cordas AB, BC, CD, DE e EF tenham os comprimentos pedidos. Agora diminua o raio do circulo ateh que A=F... Hmmm... alguem tem algum motivo para essa constr

[obm-l] Re: [obm-l] Limite:dúvida simples

2015-04-04 Por tôpico Ralph Teixeira
Se voce estah falando da SEQUENCIA a_n=sin(n.pi) (n=1,2,3,...), entao, sim, lim (n->+Inf) a_n =0, exatamente pelo motivo que voce citou. Se voce estah falando da FUNCAO f(n)=sin(n.pi) onde n eh uma variavel real, entao, nao, lim (n->Inf) f(n) nao eh ZERO. Abraco, Ralph. P.S.: O *usual* eh que n

[obm-l] Re: [obm-l] Dúvida

2015-03-29 Por tôpico Ralph Teixeira
O proprio Excel tem algo que pode ajudar. Tente isso: 1. Fazer o grafico "Scatterplot XY" dos seus dados 2. Clicar em um dos pontos que voce plotou, e selecione "Adicionar Linha de Tendencia". Voce tem que escolher o modelo (linear, exponencial), mas ele faz o resto. Se voce pedir, ele te mostra a

Re: [obm-l] Como provar?

2015-03-27 Por tôpico Ralph Teixeira
Ambos saem rápido por indução "forte". A ideia é, dada uma certa propriedade p(n), mostrar que: a) p(1) é verdadeira b) (Para k=2,3,...) se p(n) é verdadeira para n=1,2,3,...,k-1, então p(k) é verdadeira. De (a) e (b), por indução forte, conclui-se que p(n) é verdadeira para todo n natural positi

[obm-l] Re: [obm-l] É soma de dois quadrados

2015-03-10 Por tôpico Ralph Teixeira
Pelo exemplo... Hmmm... Acho que voce quis dizer: quando eh quadrado, eh QUADRADO da soma de dois quadrados? Exemplos: 8^3-7^3=13^2=(2^2+3^2)^2 105^3-104^3=181^2=(9^2+10^2)^2 1456^3-1455^3=2521^2=(35^2+36^2)^2 Eh isso? 2015-03-08 11:23 GMT-03:00 marcone augusto araújo borges < marconeborge...@ho

Re: [obm-l] Probabilidade "Random quadratic equations"

2015-03-03 Por tôpico Ralph Teixeira
p(r,s)=exp(-|r|).exp(-|s|)/4 no plano todo, por exemplo... Fiz aqui no computador, deu 1-(1/4)raiz(π)e^(1/4).(1-erf(1/2)), que eh uns 72.718%. :) 2015-03-03 13:42 GMT-03:00 Ralph Teixeira : > Impossivel responder sem que se de uma ideia da distribuicao de > probabilidade atendidas

Re: [obm-l] Probabilidade "Random quadratic equations"

2015-03-03 Por tôpico Ralph Teixeira
Impossivel responder sem que se de uma ideia da distribuicao de probabilidade atendidas por r e s (Eu sou o chato da lista que reclama que tem muito problema de probabilidade que nao tem enunciado preciso...) Uma possibilidade eh tomar r e s distribuidos uniformemente e independentemente no i

Re: [obm-l] Comprimento de Arco

2015-03-03 Por tôpico Ralph Teixeira
t^22? Serio? 2015-03-03 8:04 GMT-03:00 João Sousa : > Calcule o comprimento de arco da curva com equações paramétricas x=2t^3 e > y=4t^22 para t de 0 a 1. > > Pessoal achei 122/27, mas o gabarito é 61/9, alguém pode confirmar a > resposta? > > Abs > > João > > -- > Esta mensagem foi verificada p

Re: [obm-l] Sapato Defeituoso

2015-03-02 Por tôpico Ralph Teixeira
Como estah, o problema eh indeterminado, pois ele nao nos dah nenhuma ideia da distribuicao de probabilidade a ser atribuida aa busca do vendedor. Vejo duas alternativas mais ou menos naturais, que dao respostas diferentes, mas nenhuma delas me satisfaz: 1. HIPOTESE 1: todos os pares tem a mesma c

[obm-l] Re: [obm-l] Ajuda - OBM 2014 nível universitário

2015-02-22 Por tôpico Ralph Teixeira
Acho que a culpa dessa expressao eh minha -- eu tenho essa mania de chamar funcoes afins de "lineares", vem do ingles (linear functions). "Linear em cada entrada" quer dizer o seguinte: se voce fixar todas as entradas exceto uma, digamos, a_11=x, a funcao determinante seria f(x)=ax+b onde a e b de

[obm-l] Re: [obm-l] Equação funcional e Continuidade

2015-02-22 Por tôpico Ralph Teixeira
Tem funcoes demais... Basicamente: i) Escolha um a qualquer tal que 0: > *Prezados colegas gostaria de ajuda com o seguinte problema: > > - Encontre todas as funções contínuas f : [0,1] --> [0,1] tais que: > f(f(x)) = x . > > *Procedi da seguinte maneira: > > 1.Deduzi imediatamente (pelos fato

Re: [obm-l] Provas antigas OBM

2015-02-16 Por tôpico Ralph Teixeira
Oops! Obrigado! :) 2015-02-16 7:59 GMT-05:00 Henrique Rennó : > y=x+(a-c)/2 > > 2015-02-15 23:58 GMT-02:00 Ralph Teixeira : > >> "se" mesmo, ou "se, e somente se"? >> >> Para fazer "se": vamos multiplicar por 4 e completar quadrados: &

Re: [obm-l] Provas antigas OBM

2015-02-15 Por tôpico Ralph Teixeira
"se" mesmo, ou "se, e somente se"? Para fazer "se": vamos multiplicar por 4 e completar quadrados: (2x+a)^2+(4b-a^2)=(2y+c)^2+(4d-c^2) Agora, se a^2-4b=c^2-4d, ficamos com (2x+a)^2=(2y+c)^2 que claramente tem infinitas solucoes inteiras do tipo 2x+a=2y+c; de fato, basta tomar x inteiro qualque

Re: [obm-l] Provar que y tem uma infinidade de zeros em R

2015-02-10 Por tôpico Ralph Teixeira
Este problema já apareceu aqui na lista, mas acho que ninguém resolveu a contento. Então vou dar meu palpite. Seja M o ínfimo positivo de g(x), isto é, g(x)>=M>0 para todo x real. ---///--- Espírito da demonstração: a) Se y for positiva e estiver descendo, a EDO faz y descer cada vez mais rápido,

[obm-l] Re: [obm-l] Questão de probabilidade (reformulação)

2015-01-18 Por tôpico Ralph Teixeira
Ainda existe o problema: na hora de escolher uma solucao (x,y,z), qual a distribuicao de probabilidade a ser utilizada? Mas, agora que voce restringiu o problema, a interpretacao mais natural leva aa mesma solucao daquele de "dividir uma vareta" que eu pus no meu link, de uma olhada lah. Abraco, R

[obm-l] Re: [obm-l] Re: [obm-l] Questão de probabilidade

2015-01-18 Por tôpico Ralph Teixeira
Eh, o Hermann tem razao, nao existe uma distribuicao de probabilidade nos reais positivos que "funcione" bem. O problema eh que existem varias maneiras de "escolher um numero real positivo aleatoriamente", nenhuma delas completamente padrao, e elas dariam respostas diferentes para seu problema. ---

[obm-l] Re: [obm-l] Limite de sequência

2015-01-16 Por tôpico Ralph Teixeira
Para ser chato, todas as frases abaixo estao corretas no universo dos Reais: "x^x^x^x...=2 IMPLICA x=raiz(2)" "x^x^x^x...=4 IMPLICA x=raiz(2)" "x^2+4=0 IMPLICA x=2" "x^2+4=0 IMPLICA x=13" "2x+x-3x=25 IMPLICA x=755" "2x+x-3x=25 IMPLICA que eu sou o Papa" (O problema eh entender o que significa a pa

Re: [obm-l] Desigualdade

2015-01-15 Por tôpico Ralph Teixeira
Se voce nao quiser usar Taylor, pode fazer assim (que no fundo no fundo eh Taylor disfarcado): Seja f(x)=e^x-1-x-x^2/4. Note que f'(x)=e^x-1-x/2 e f''(x)=e^x-1/2 Como f''(x)>0 para todo x>0, temos que f'(x) eh crescente em (0,+Inf). Como f'(0)=0, isto significa que f'(x)>0 em (0,+Inf). Entao f(x

Re: [obm-l] Integral interessante

2015-01-07 Por tôpico Ralph Teixeira
Gostei, bem bonitinho! Primeiro faremos x=az onde 0: > Para a > 0, determinar > > I(a) = Int (0, oo) ln(x)/(x^2 + a^2) > > Abraços. > > Artur Costa Steiner > -- > Esta mensagem foi verificada pelo sistema de antivírus e > acredita-se estar livre de perigo. > > >

Re: [obm-l] Inteiros positivos

2014-12-25 Por tôpico Ralph Teixeira
Ficou subentendido que m e n sao naturais positivos. n=1 nao serve, entao o lado direito eh par. Entao m eh impar, digamos, m=2k+1. Entao fica n.2^(n-1)=4k(k+1). Como n=2 nao serve, podemos escrever n.2^(n-3)=k(k+1). Note que n=4 nao serve, e n=5 dah aquela solucao. Agora, o problema eh que um d

[obm-l] Re: [obm-l] Sequência Complicada

2014-12-19 Por tôpico Ralph Teixeira
Bom, esses problemas de "termo geral" sao esquisitos... Eh mais facil ver COMO A SEQUENCIA FOI GERADA para adivinhar o termo geral! Por exemplo, eu chuto que sua sequencia veio de uma recorrencia assim (este tipo de coisa aparece muito quando voce estah resovendo EDOs por Series de Potencias): a_

Re: [obm-l] Soma de Quadrados

2014-12-19 Por tôpico Ralph Teixeira
Ah, achei um errinho de sinal... :( Deixa eu tentar de novo: Note que dah para escrever m de forma mais explicita. m=[n^2-(n-1)^2]+[(n-2)^2-(n-3)^2]+...+[(n-(k-1))^2-(n-k)^2] onde tem (k+1)/2 pares de termos ali em cima m=[2n-1]+[2n-5]+[2n-9]+...+[2n-(2k-1)] m=n(k+1)-[1+5+9+...+(2k-1)] m=n(k+1)-

Re: [obm-l] Soma de Quadrados

2014-12-19 Por tôpico Ralph Teixeira
Consigo arrumar o problema, mas vai ficar faltando um pedaco no final. Note que dah para escrever m de forma mais explicita. m=[n^2-(n-1)^2]+[(n-2)^2-(n-3)^2]+...+[(n-(k-1))^2-(n-k)^2] onde tem (k+1)/2 pares de termos ali em cima m=[2n-1]+[2n-5]+[2n-9]+...+[2n-(2k-1)] m=n(k+1)-[1+5+9+...+(2k-1)]

Re: [obm-l] Derivadas parciais

2014-12-19 Por tôpico Ralph Teixeira
-02:00 Ralph Teixeira : > >> 1) Supondo que o dominio eh R^2: se a derivada de algo com relacao a x eh >> zero, entao essa coisa nao depende de x, certo? >> >> Entao se d2f/dxdy=0, isto significa que df/dy=h(y), onde h(y) eh uma >> funcao qualquer que soh depende de y. &

Re: [obm-l] Derivadas parciais

2014-12-17 Por tôpico Ralph Teixeira
1) Supondo que o dominio eh R^2: se a derivada de algo com relacao a x eh zero, entao essa coisa nao depende de x, certo? Entao se d2f/dxdy=0, isto significa que df/dy=h(y), onde h(y) eh uma funcao qualquer que soh depende de y. Agora integre isso: f(x,y)=Int h(y) dy = H(y)+C onde H eh uma anti-d

Re: [obm-l] Como provar?

2014-12-06 Por tôpico Ralph Teixeira
Hmmm, nao. Por exemplo, se n=4, poderiam ser vertices de um retangulo. 2014-12-06 15:50 GMT-02:00 Artur Steiner : > Aliás, por um raciocínio similar, isto pode ser generalizado para n > complexos. Seus afixos formam um n-ágono regular convexo. > > Artur Costa Steiner > > Em 06/12/2014, às 14:38,

[obm-l] Re: [obm-l] Sequência

2014-11-19 Por tôpico Ralph Teixeira
Ah, pequena correcao, esqueci um "1-". Devia ser: X_n = 1 - 2raiz(3)/3 . | sin[(n-1).pi/3] |. 2014-11-19 20:42 GMT-02:00 Ralph Teixeira : > Vou supor que suas sequencias comecam do indice 1, e nao do indice 0. > > 1) Dado k fixo, tome Y_n=X_(n+k-1) (n=1,2,3,...) > > 2)

<    1   2   3   4   5   6   7   8   >